LSAT and Law School Admissions Forum

Get expert LSAT preparation and law school admissions advice from PowerScore Test Preparation.

 hfsports429
  • Posts: 12
  • Joined: Dec 10, 2016
|
#31923
Hi, I was just wondering the difference between answer choices A and E. I was having a tough time picking between them and went with E since the author seemed (to me) to be refuting the commentators' arguments with logical proof and discussion. I get why A would be write, but could someone explain why E is wrong?

Thanks.
 Kristina Moen
PowerScore Staff
  • PowerScore Staff
  • Posts: 230
  • Joined: Nov 17, 2016
|
#31954
Hi hfsports,

Keep in mind that we are being asked about what the author thinks that the commentators believe. Yes, the author thinks that the commentator's implications are not logical (aka do not logically follow) from the theory of philosophical anarchism. But the author uses the word "counterintuitive" to describe what the commentator's believe. And they do NOT believe those implications contain an internal logical inconsistency - in fact, they think those implications follow logically! But the underlying assumption here is that those further conclusions conflict with commonly held beliefs. Otherwise, why would the commentators bring them up?

An internal logical inconsistency would be when two claims cannot be true at the same time. So an argument that contains an internal logical inconsistency is one that relies on premises that cannot both be true, or the conclusion and the premise cannot both be true. For example: "Everyone should join our country club. After all, it's an exclusive club!"
 bk1111
  • Posts: 103
  • Joined: Apr 22, 2017
|
#46685
Hello - I have been going in circles between A and C. I don't really understand what C is saying and therefore why it's incorrect. Can someone flush this out for me please?
 Adam Tyson
PowerScore Staff
  • PowerScore Staff
  • Posts: 5153
  • Joined: Apr 14, 2011
|
#47283
I'll try , bk1111!

To understand these answers better, try substituting them back into the passage in place of the language we are being asked about, that those implications are counterintuitive. Here's what that would look like with answer A, the correct answer:

"Some commentators have rejected this position because [it conflicts with some commonly held beliefs]"

Sounds about right. What about plugging in answer C instead?

"Some commentators have rejected this position because [common sense indicates that philosophical anarchism does not have the implications that follow]"

That's the opposite of what the author is getting at. He is trying to say that those commentators think philosophical anarchism HAS two implications (the ones he discusses just after those lines), and that those implications are counterintuitive, meaning they conflict with what our intuition tells us should be true. Answer C says the commentators believe philosophical anarchism could not reasonably be expected to have those implications.

It's a pretty messy answer, and I hope my scrubbing it made it look a little cleaner for you! ;-)
 J1445
  • Posts: 11
  • Joined: Aug 05, 2020
|
#79043
Right. Ok, that makes sense. Thank you!

Get the most out of your LSAT Prep Plus subscription.

Analyze and track your performance with our Testing and Analytics Package.